Math modelling diffrential equation question?

Click For Summary
The discussion revolves around solving a differential equation related to the deterioration of a product in a container. The equation dx/dt = -kx describes the rate of deterioration, leading to the general solution x = A e^(-kt). The user successfully solved the first two parts, showing that at t=1, the amount of product is Pq, and at t=2, they need to determine the total amount after adding more product. The key insight provided clarifies that the amounts added at different times decay according to their respective time delays, leading to the final expression of P(q^2 + q + 1) after t=2. This understanding helps resolve the user's confusion about how to account for the changes in product amount over time.
Eveflutter
Messages
14
Reaction score
0
So there's this question I've to do. I got through until a certain point and now I'm stuck. >->

1&2The question:
The rate of deterioration of a product in a container is proportional to the amount of product present. At time t, the amount of product is x.
(i) State the diffrential equation relating x and t and solve the general solution to show that x=Ae-kt where A is an arbitrary contant and k is the contant of proportionality.
(ii) Before t=0, no product was present in the container. At t=0, P amount of the product was added to the container. When t=1, the amount of product in the container was Pq, where q is a constant such that 0<q<1. Show that x=Pqt.
(iii) When t=1 and again when t=2, another P amount of x was added to the container. Show that the amount of product in the container immediately after t=2 was P(q2+q+1)

3I got through with the first two parts:
(i) dx/dt=-kx
-> x=e-kt+c
-> x=Ae-kt , A=ec
(ii)when t=0, x=P
-> A=P
-> x= Pe-kt
when t=1, x=Pq
-> Pq=Pe-kt
-> q=e-kt
-> x=Pqt

I'm stuck at part (iii)
I tried reasoning it out but I don't know. Would the equation x=Pqt change since the intials are now changing twice (at t=1 and t=2)? But if so, how would I get the new ones from tha information?

Alright so as t approahes 1, x tends to Pq. At t=1 the amount of product changes to Pq+P. Then as t approaches 2, x deteriorates from Pq+P. But by how much?? Let's say it deteriorates till an amount B? Now at t=2, the amount of product goes to B+P.
I guess the P part of the required result (P(q2+q+1)) is that P in B+P. How do i get B?

I would really appreciate any help on this question, thank you!
 
Physics news on Phys.org
Eveflutter said:
So there's this question I've to do. I got through until a certain point and now I'm stuck. >->

1&2The question:
The rate of deterioration of a product in a container is proportional to the amount of product present. At time t, the amount of product is x.
(i) State the diffrential equation relating x and t and solve the general solution to show that x=Ae-kt where A is an arbitrary contant and k is the contant of proportionality.
(ii) Before t=0, no product was present in the container. At t=0, P amount of the product was added to the container. When t=1, the amount of product in the container was Pq, where q is a constant such that 0<q<1. Show that x=Pqt.
(iii) When t=1 and again when t=2, another P amount of x was added to the container. Show that the amount of product in the container immediately after t=2 was P(q2+q+1)

3I got through with the first two parts:
(i) dx/dt=-kx
-> x=e-kt+c
-> x=Ae-kt , A=ec
(ii)when t=0, x=P
-> A=P
-> x= Pe-kt
when t=1, x=Pq
-> Pq=Pe-kt
-> q=e-kt
-> x=Pqt

I'm stuck at part (iii)
I tried reasoning it out but I don't know. Would the equation x=Pqt change since the intials are now changing twice (at t=1 and t=2)? But if so, how would I get the new ones from tha information?

Alright so as t approahes 1, x tends to Pq. At t=1 the amount of product changes to Pq+P. Then as t approaches 2, x deteriorates from Pq+P. But by how much?? Let's say it deteriorates till an amount B? Now at t=2, the amount of product goes to B+P.
I guess the P part of the required result (P(q2+q+1)) is that P in B+P. How do i get B?

I would really appreciate any help on this question, thank you!

Think of it this way. The amount ##P## at ##t=0## decreases according to ##Pq^t##. The amount ##P## added at ##t=1## decreases like ##Pq^{t-1}## (it's just 1 second delayed from the first). Then at ##t=2## you add ##Pq^{t-2}##. So the total after ##t=2## is ##Pq^t+Pq^{t-1}+Pq^{t-2}##. Correct?
 
Dick said:
Think of it this way. The amount ##P## at ##t=0## decreases according to ##Pq^t##. The amount ##P## added at ##t=1## decreases like ##Pq^{t-1}## (it's just 1 second delayed from the first). Then at ##t=2## you add ##Pq^{t-2}##. So the total after ##t=2## is ##Pq^t+Pq^{t-1}+Pq^{t-2}##. Correct?
Ooooh! I understood what you were doing but not why you were doing it but it took me a couple minutes to completely get it. Thank you so much! I really appreicate your answer :)
 
Question: A clock's minute hand has length 4 and its hour hand has length 3. What is the distance between the tips at the moment when it is increasing most rapidly?(Putnam Exam Question) Answer: Making assumption that both the hands moves at constant angular velocities, the answer is ## \sqrt{7} .## But don't you think this assumption is somewhat doubtful and wrong?

Similar threads

Replies
5
Views
2K
  • · Replies 1 ·
Replies
1
Views
1K
  • · Replies 24 ·
Replies
24
Views
3K
  • · Replies 7 ·
Replies
7
Views
1K
  • · Replies 7 ·
Replies
7
Views
2K
Replies
6
Views
2K
Replies
3
Views
2K
  • · Replies 12 ·
Replies
12
Views
3K
  • · Replies 6 ·
Replies
6
Views
1K
Replies
2
Views
1K